Đến nội dung

Hình ảnh

Topic tổng hợp một số bất đẳng thức trong kì thi MO các nước

* * * * * 16 Bình chọn

  • Please log in to reply
Chủ đề này có 501 trả lời

#301
Nguyen Minh Hai

Nguyen Minh Hai

    Thiếu úy

  • Thành viên
  • 666 Bài viết

Bài 111(Greece MO)(Nguyễn Duy Khương): Cho a,b,c là các số thực dương sao cho ab+bc+ca=1 . Cmr:

 

                           

                          $\sqrt{\frac{a^2+1}{b^2+6bc+c^2}}+\sqrt{\frac{b^2+1}{c^2+6ca+a^2}}+\sqrt{\frac{c^2+1}{a^2+6ab+b^2}}\geq \frac{3\sqrt{2}}{2}$

Áp dụng BĐT $AM-GM$ ta có:

$\sum \sqrt{\frac{a^2+1}{b^2+6bc+c^2}}=\sum \sqrt{\frac{a^2+ab+bc+ca}{(b+c)^2+4bc}}$

 

$=\sum \sqrt{\frac{(a+b)(c+a)}{(b+c)^2+4bc}}$

 

$ \geq \sum \sqrt{\frac{(a+b)(c+a)}{2(b+c)^2}}$

 

$\geq 3.\sqrt[6]{\frac{(a+b)^2(b+c)^2(c+a)^2}{8(a+b^2)(b+c)^2(c+a)^2}}=\frac{3\sqrt{2}}{2}$ 

Xảy ra dấu $=$ khi $a=b=c=\frac{1}{\sqrt{3}}$


Bài viết đã được chỉnh sửa nội dung bởi khanghaxuan: 18-06-2015 - 18:32


#302
dangkhuong

dangkhuong

    Sĩ quan

  • Thành viên
  • 312 Bài viết

Bài 112:(Canada MO): Cho a,b,c>0 sao cho a+b+c=3. Cmr: 

 

$\frac{a}{\sqrt{a^3+a^2+1}}+\frac{b}{\sqrt{b^3+b^2+1}}+\frac{c}{\sqrt{c^3+c^2+1}}\leq \sqrt{3}$


Bài viết đã được chỉnh sửa nội dung bởi khanghaxuan: 18-06-2015 - 18:35

:ukliam2:  :ukliam2:  :ukliam2:


#303
dangkhuong

dangkhuong

    Sĩ quan

  • Thành viên
  • 312 Bài viết

Bài 113:(Palestine MO): Cho a,b,c>0. Cmr:

 

$\frac{(a^3+2abc)^3}{(2a^3+c^3)^2}+\frac{(b^3+2abc)^3}{(2b^3+c^3)^2}+\frac{(c^3+2abc)^3}{(2c^3+a^3)^2}\leq 3(a^3+b^3+c^3)$


Bài viết đã được chỉnh sửa nội dung bởi khanghaxuan: 18-06-2015 - 18:33

:ukliam2:  :ukliam2:  :ukliam2:


#304
Hoang Nhat Tuan

Hoang Nhat Tuan

    Hỏa Long

  • Thành viên
  • 974 Bài viết

Bài 112:(Canada MO)(Nguyễn Duy Khương): Cho a,b,c>0 sao cho a+b+c=3. Cmr: :ukliam2:  :ukliam2:  :ukliam2: 

 

$\frac{a}{\sqrt{a^3+a^2+1}}+\frac{b}{\sqrt{b^3+b^2+1}}+\frac{c}{\sqrt{c^3+c^2+1}}\leq \frac{\sqrt{3}}{3}$

Đề sai rồi, với $a=b=c=1$ thì VT>VT

VP phải là $\sqrt{3}$ chứ :(


Ngài có thể trói cơ thể tôi, buộc tay tôi, điều khiển hành động của tôi: ngài mạnh nhất, và xã hội cho ngài thêm quyền lực; nhưng với ý chí của tôi, thưa ngài, ngài không thể làm gì được.

#305
nangcuong8e

nangcuong8e

    Trung sĩ

  • Thành viên
  • 134 Bài viết

Bài 112:(Canada MO): Cho a,b,c>0 sao cho a+b+c=3. Cmr: 

 

$\frac{a}{\sqrt{a^3+a^2+1}}+\frac{b}{\sqrt{b^3+b^2+1}}+\frac{c}{\sqrt{c^3+c^2+1}}\leq \sqrt{3}$

Ta có: $\sum \frac{a}{\sqrt{a^3 +a^2+1}} \leq \sqrt{3(\sum \frac{a^2}{a^3 +a^2 +1})}$ 
 Do đó ta cần chứng minh $\sum \frac{a^2}{a^3 +a^2+1} \leq 1$

Hay cụ thể ta phải chứng minh $\frac{a^2}{a^3+a^2+1} \leq \frac{a+2}{9}$ (1)

 Thật vậy, (1) $\Leftrightarrow a^4 +3a^3 -7a^2 +a +2 \geq 0$

$\Leftrightarrow (a-1)^2(a^2+5a+2) \geq 0$ (luôn đúng với $a >0$)

$\Rightarrow \sum \frac{a^2}{a^3 +a^2 +1} \leq \sum \frac{a+2}{9} = \frac{3 +2.3}{9} =1$

 Dấu $"="$ xảy ra khi $a=b=c=1$


Bài viết đã được chỉnh sửa nội dung bởi nangcuong8e: 18-06-2015 - 19:14


#306
Belphegor Varia

Belphegor Varia

    Thượng sĩ

  • Thành viên
  • 227 Bài viết

Bài 114 (VMO 1999) : Xét các số thực dương $a,b$ sao cho phương trình : 

                                                                                     $ax^{3}-x^{2}+bx-1=0$

có ba nghiệm thực dương ( các nghiệm có thể bằng nhau).

 

Tìm giá trị nhỏ nhất của biểu thức $P=\frac{5a^{2}-3ab+2}{a^{2}(b-a)}$ .


$ \textbf{NMQ}$

Wait a minute, You have enough time. Also tomorrow will come 

Just take off her or give me a ride 

Give me one day or one hour or just one minute for a short word 

 


#307
Hoang Tung 126

Hoang Tung 126

    Thiếu tá

  • Thành viên
  • 2061 Bài viết

Bài 114 (VMO 1999) : Xét các số thực dương $a,b$ sao cho phương trình : 

                                                                                     $ax^{3}-x^{2}+bx-1=0$

có ba nghiệm thực dương ( các nghiệm có thể bằng nhau).

 

Tìm giá trị nhỏ nhất của biểu thức $P=\frac{5a^{2}-3ab+2}{a^{2}(b-a)}$ .

 Gọi $x_{1},x_{2},x_{3}$ là 3 nghiệm dương của phương trình. 

   Theo định lý Viet cho phương trình bậc 3 ta có :

 

  $\left\{\begin{matrix} x_{1}x_{2}+x_{2}x_{3}+x_{3}x_{1}=\frac{b}{a} & & \\ x_{1}+x_{2}+x_{3}=\frac{1}{a} & & \\ x_{1}x_{2}x_{3}=\frac{1}{a} & & \end{matrix}\right.$

 

Ta có :$\frac{b}{a^2}=\frac{b}{a}.\frac{1}{a}=(\sum x_{1}x_{2})(\sum x_{1})$

           $\frac{\sum x_{1}x_{2}}{x_{1}x_{2}x_{3}}=\frac{\frac{b}{a}}{\frac{1}{a}}=b= > \sum \frac{1}{x_{1}}=b= > \frac{b}{a}=(\sum \frac{1}{x_{1}})(\sum x_{1})$

( Do $\sum x_{1}=\frac{1}{a}$)

Theo Cosi ta có :$\frac{1}{a}=x_{1}x_{2}x_{3}=\sum x_{1}\geq 3\sqrt[3]{x_{1}x_{2}x_{3}}= > (x_{1}x_{2}x_{3})^3\geq 27x_{1}x_{2}x_{3}= > x_{1}x_{2}x_{3}\geq \sqrt{27}=3\sqrt{3}$

 

  Từ đó 

$P=\frac{5a^2-3ab+2}{a^2(b-a)}$

$=\frac{\frac{5a^2-3ab+2}{a^3}}{\frac{a^2(b-a)}{a^3}}=\frac{\frac{5}{a}-\frac{3b}{a^2}+\frac{2}{a^3}}{\frac{b}{a}-1}$

$=\frac{5x_{1}x_{2}x_{3}-3(\sum x_{1})(\sum x_{1}x_{2})+2(\sum x_{1})^3}{(\sum x_{1})(\sum \frac{1}{x_{1}})-1}$

$=(x_{1}x_{2}x_{3})(\frac{5x_{1}x_{2}x_{3}-3(\sum x_{1})(\sum x_{1}x_{2})+2(\sum x_{1})^3}{(\sum x_{1})(\sum x_{1}x_{2})-x_{1}x_{2}x_{3}})$

$\geq 3\sqrt{3}.(\frac{5x_{1}x_{2}x_{3}-3\sum x_{1}x_{2}(x_{1}+x_{2})-9x_{1}x_{2}x_{3}+2(\sum x_{1})^3}{\sum x_{1}x_{2}(x_{1}+x_{2})+2x_{1}x_{2}x_{3}})$

$= > P\geq 3\sqrt{3}.\left [ \frac{-4x_{1}x_{2}x_{3}-3\sum x_{1}x_{2}(x_{1}+x_{2})+2(\sum x_{1})^3}{\sum x_{1}x_{2}(x_{1}+x_{2})+2x_{1}x_{2}x_{3}} \right ]$

 

Ta sẽ chứng minh:   $P\geq 12\sqrt{3}$

$< = > \frac{-4x_{1}x_{2}x_{3}-3\sum x_{1}x_{2}(x_{1}+x_{2})+2(\sum x_{1})^3}{\sum x_{1}x_{2}({x_{1}+x_{2}})+2x_{1}x_{2}x_{3}}\geq 4$

$< = > 2(\sum x_{1})^3-3\sum x_{1}x_{2}(x_{1}+x_{2})-4x_{1}x_{2}x_{3}\geq 4\sum x_{1}x_{2}(x_{1}+x_{2})+8x_{1}x_{2}x_{3}$

$< = > 2\sum x_{1}^3+6\sum x_{1}x_{2}(x_{1}+x_{2})+12x_{1}x_{2}x_{3}-3\sum x_{1}x_{2}(x_{1}+x_{2})-4x_{1}x_{2}x_{3}$

$\geq 4\sum x_{1}x_{2}(x_{1}+x_{2})+8x_{1}x_{2}x_{3}$

$< = > 2\sum x_{1}^3\geq \sum x_{1}x_{2}(x_{1}+x_{2})$

$< = > \sum (x_{1}+x_{2})(x_{1}-x_{2})^2\geq 0$

 ( Điều này luôn đúng với $x_{1},x_{2},x_{3}> 0$)

 

  Do đó $P\geq 12\sqrt{3}= > P_{min}=12\sqrt{3}< = > x_{1}=x_{2}=x_{3}< = > a=\frac{1}{3\sqrt{3}},b=\sqrt{3}$


Bài viết đã được chỉnh sửa nội dung bởi Hoang Tung 126: 18-06-2015 - 22:22


#308
Belphegor Varia

Belphegor Varia

    Thượng sĩ

  • Thành viên
  • 227 Bài viết

Bài 115 (CĐTMO 2006) : Chứng minh rằng với mọi số thực $x,y,z$ thuộc đoạn $[1;2]$ ta luôn có bất đẳng thức :
                                          $(x+y+z)(\frac{1}{x}+\frac{1}{y}+\frac{1}{z})\geq 6(\frac{x}{y+z}+\frac{y}{x+z}+\frac{z}{x+y}).$

                                           Hỏi đẳng thức xảy ra khi và chỉ khi nào ? 

Bài 116 (CĐTMO 2005) : Chứng minh rằng
                                                                 $\frac{a^{3}}{(b+c)^{3}}+\frac{b^{3}}{(c+a)^{3}}+\frac{c^{3}}{(a+b)^{3}}$ $\geq \frac{3}{8}$

                                        trong đó $a,b,c$ là các số dương.

Bài 117 (CĐTMO 2001) : Xét các số thực dương $a,b,c$ thỏa mãn điều kiện 

                                                                          $21ab+2bc+8ca\leq 12$
                                        Tìm giá trị nhỏ nhất của biểu thức $P=\frac{1}{a}+\frac{2}{b}+\frac{3}{c}.$ 

 

Spoiler


 


Bài viết đã được chỉnh sửa nội dung bởi Belphegor Varia: 19-06-2015 - 14:36

$ \textbf{NMQ}$

Wait a minute, You have enough time. Also tomorrow will come 

Just take off her or give me a ride 

Give me one day or one hour or just one minute for a short word 

 


#309
Nguyen Minh Hai

Nguyen Minh Hai

    Thiếu úy

  • Thành viên
  • 666 Bài viết

 

Bài 117 (CĐTMO 2001) : Xét các số thực dương $a,b,c$ thỏa mãn điều kiện 

                                                                          $21ab+2bc+8ca\leq 12$
                                        Tìm giá trị nhỏ nhất của biểu thức $P=\frac{1}{a}+\frac{2}{b}+\frac{3}{c}.$ 

 

Spoiler


 

Bài toán này đã có khá nhiều cách giải trên diễn đàn

http://diendantoanho...-frac2b-frac3c/

http://diendantoanho...c3cgeq-frac152/

Thêm một cách mà mình vừa nghĩ ra  :icon10: 

Từ điểm rơi của bài toán là $(a,b,c)=\left ( \frac{1}{3};\frac{4}{5};\frac{3}{2} \right )$

$\Rightarrow 36a=15b=8c=12$

Do đó ta đặt: $\left\{\begin{matrix} 36a=x & & & \\ 15b=y & & & \\ 8c=z & & & \end{matrix}\right.$

Bài toán trở thành: 

Cho $x,y,z>0$ thõa mãn: $\frac{7xy}{180}+\frac{yz}{60}+\frac{zx}{36} \leq 12$.TÌm GTNN của:

$P=\frac{36}{x}+\frac{30}{y}+\frac{24}{z}$

Từ giả thiết ta có: 

$2160 \geq 7xy+3yz+5zx \geq 15\sqrt[15]{x^7y^7.y^3z^3.z^5x^5}$

$\Rightarrow x^6y^5z^4 \leq 12^5$            (Theo $AM-GM$)

Do đó: $P=6(\frac{6}{x}+\frac{5}{y}+\frac{4}{z})\geq 6.15\sqrt[15]{\frac{1}{x^6y^5z^4}} \geq 90.\sqrt[15]{\frac{1}{12^{15}}}=\frac{15}{2}$

$\Rightarrow GTNN_{P}=\frac{15}{2}$ khi $(a,b,c)=\left ( \frac{1}{3};\frac{4}{5};\frac{3}{2} \right )$

[spoiler] Đề thi vòng 16 Vio 9 năm nay cũng ra bài này [\spoiler]


Bài viết đã được chỉnh sửa nội dung bởi Nguyen Minh Hai: 19-06-2015 - 09:47


#310
Nguyen Minh Hai

Nguyen Minh Hai

    Thiếu úy

  • Thành viên
  • 666 Bài viết

 

Bài 116 (CĐTMO 2005) : Chứng minh rằng
                                                                 $\frac{a^{3}}{(b+c)^{3}}+\frac{b^{3}}{(c+a)^{3}}+\frac{c^{3}}{(a+b)^{3}}$

                                        trong đó $a,b,c$ là các số dương.

 

 

Có vẽ đề bị thiếu thì phải?  :closedeyes:



#311
dogsteven

dogsteven

    Đại úy

  • Thành viên
  • 1567 Bài viết

Bài toán này đã có khá nhiều cách giải trên diễn đàn

http://diendantoanho...-frac2b-frac3c/

http://diendantoanho...c3cgeq-frac152/

Thêm một cách mà mình vừa nghĩ ra  :icon10: 

Từ điểm rơi của bài toán là $(a,b,c)=\left ( \frac{1}{3};\frac{4}{5};\frac{3}{2} \right )$

$\Rightarrow 36a=15b=8c=12$

Do đó ta đặt: $\left\{\begin{matrix} 36a=x & & & \\ 15b=y & & & \\ 8c=z & & & \end{matrix}\right.$

Bài toán trở thành: 

Cho $x,y,z>0$ thõa mãn: $\frac{7xy}{180}+\frac{yz}{60}+\frac{zx}{36} \leq 12$.TÌm GTNN của:

$P=\frac{36}{x}+\frac{30}{y}+\frac{24}{z}$

Từ giả thiết ta có: 

$2160 \geq 7xy+3yz+5zx \geq 15\sqrt[15]{x^7y^7.y^3z^3.z^5x^5}$

$\Rightarrow x^6y^5z^4 \leq 12^5$            (Theo $AM-GM$)

Do đó: $P=6(\frac{6}{x}+\frac{5}{y}+\frac{4}{z})\geq 6.15\sqrt[15]{\frac{1}{x^6y^5z^4}} \geq 90.\sqrt[15]{\frac{1}{12^{15}}}=\frac{15}{2}$

$\Rightarrow GTNN_{P}=\frac{15}{2}$ khi $(a,b,c)=\left ( \frac{1}{3};\frac{4}{5};\frac{3}{2} \right )$

[spoiler] Đề thi vòng 16 Vio 9 năm nay cũng ra bài này [\spoiler]

Lời giải không quan trọng, tìm được điểm rơi mới là điều tất yếu để giải quyết bài toán.

Không biết có cái hệ nào gọn hơn không chứ AM-GM suy rộng và Lagrange đều ra cái hệ như thế này:

$6yz-1=6zx-4=4xy-7$ và $2x+8y+21z=12xyz$ với $ax=by=cz=1$

Tuy nhiên, vẫn thích cách thế trực tiếp hơn, vừa tự nhiên, vừa đỡ tốn sức tìm điểm rơi.


Bài viết đã được chỉnh sửa nội dung bởi dogsteven: 19-06-2015 - 12:35

Quyết tâm off dài dài cày hình, số, tổ, rời rạc.


#312
dogsteven

dogsteven

    Đại úy

  • Thành viên
  • 1567 Bài viết

Có vẽ đề bị thiếu thì phải?  :closedeyes:

Chắc là chứng minh nó lớn hơn hoặc bằng $\dfrac{3}{8}$ :))


Quyết tâm off dài dài cày hình, số, tổ, rời rạc.


#313
dogsteven

dogsteven

    Đại úy

  • Thành viên
  • 1567 Bài viết

Bài 115. 

Bất đẳng thức đã cho tương đương với: $\sum \left[\dfrac{1}{yz}-\dfrac{3}{(x+y)(x+z)}\right](y-z)^2\geqslant 0$

(1) $(y+z)(y+x)-3zx=y^2+y(z+x)-2zx=y(y+z)+xy-2zx\geqslant 2x(y-z)\geqslant 0$

(2) $\dfrac{1}{yz}-\dfrac{3}{(x+y)(x+z)}\geqslant \dfrac{1}{4yz}\geqslant 0$

(3) $\dfrac{1}{zx}+\dfrac{1}{xy}-\dfrac{3}{(y+z)(y+x)}-\dfrac{3}{(z+x)(z+y)}=\dfrac{x^2(y^2+z^2)+x(y^3+z^3)+yz(y+z)^2-4x^2yz}{xyz(x+y)(y+z)(z+x)}$

Mà $x^2(y^2+z^2)+x(y^3+z^3)+yz(y+z)^2-4x^2yz\geqslant x(y^3+z^3)-x^2yz\geqslant yz(y+z-x)\geqslant 0$

nên $\dfrac{1}{zx}+\dfrac{1}{xy}-\dfrac{3}{(y+z)(y+x)}-\dfrac{3}{(z+x)(z+y)}\geqslant 0$

Theo tiêu chuẩn S.O.S ta có điều phải chứng minh.


Bài viết đã được chỉnh sửa nội dung bởi dogsteven: 19-06-2015 - 13:01

Quyết tâm off dài dài cày hình, số, tổ, rời rạc.


#314
Nguyenhuyen_AG

Nguyenhuyen_AG

    Trung úy

  • Thành viên nổi bật 2016
  • 945 Bài viết
Bài 116 (CĐTMO 2005) : Chứng minh rằng

                                                                 $\frac{a^{3}}{(b+c)^{3}}+\frac{b^{3}}{(c+a)^{3}}+\frac{c^{3}}{(a+b)^{3}}$

                                        trong đó $a,b,c$ là các số dương.

 

Đề bài này sai rồi, đề đúng là như vầy

\[\frac{a^{3}}{(a+b)^{3}}+\frac{b^{3}}{(b+c)^{3}}+\frac{c^{3}}{(c+a)^{3}} \geqslant \frac{3}{8}.\]

Tác giả là thầy Nam Dũng.

 

Lời giải 1. Sử dụng bất đẳng thức Cauchy-Schwarz, ta có \[\left [ \sum \left ( \frac{a}{a+b} \right )^3 \right ]\left [ \sum c^3(a+b)^3 \right ] \ge \left (\sum c^{\frac{3}{2}}a^{\frac{3}{2}} \right )^2,\] như vậy ta cần chứng minh được \[8\left (\sum c^{\frac{3}{2}}a^{\frac{3}{2}}\right )^2 \ge 3 \sum c^3(a+b)^3 .\] Đặt $x=\sqrt{ab},\,y=\sqrt{bc},\,z=\sqrt{ca},$ bất đẳng thức trên trở thành \[8\left (\sum z^3\right )^2 \ge 3 \sum (y^2+z^2)^3,\] hay là \[\sum (x^6+y^6)+16\sum x^3y^3 \ge 9 \sum x^2y^2(x^2+y^2),\] bất đẳng thức này đúng vì \[x^6+y^6+16x^3y^3-9x^2y^2(x^2+y^2) =(x-y)^4(x^2+4xy+y^2) \ge 0.\] Chứng minh hoàn tất.

 

Lời giải 2. Theo bất đẳng thức trung bình lũy thừa, ta có \[\sqrt[3]{\frac{\displaystyle\sum \left ( \frac{a}{a+b} \right )^3}{3}}\ge\sqrt{\frac{\displaystyle\sum \left ( \frac{a}{a+b} \right )^2}{3}}.\] Như vậy, để hoàn tất chứng minh thì ta cần chỉ ra được \[\left ( \frac{a}{a+b} \right )^2+\left ( \frac{b}{b+c} \right )^2+\left ( \frac{c}{c+a} \right )^2 \ge \frac{3}{4}.\] Thay $\left ( \frac{b}{a},\,\frac{c}{b},\,\frac{a}{c} \right )$ bởi $(x,\,y,\,z)$ ta sẽ đưa bài toán về chứng minh \[\frac{1}{(x+1)^2}+\frac{1}{(y+1)^2}+\frac{1}{(z+1)^2}\ge\frac{3}{4},\] với điều kiện $xyz=1.$

 

Sử dụng bất đẳng thức Cauchy-Schwarz, ta có \[\frac{1}{(x+1)^2}+\frac{1}{(y+1)^2}\ge \frac{1}{(\frac{x}{y}+1)(xy+1)}+\frac{1}{(\frac{y}{x}+1)(xy+1)}=\frac{1}{xy+1}.\] Ta sẽ chứng minh \[\frac{1}{xy+1}+\frac{1}{(z+1)^2}\ge \frac{3}{4},\] hay là \[\frac{z}{1+z}+\frac{1}{(z+1)^2}\ge \frac{3}{4}.\] Bất đẳng thức này đúng bởi vì \[\frac{z}{1+z}+\frac{1}{(z+1)^2}-\frac{3}{4}=\frac{(z-1)^2}{4(z+1)} \ge 0.\]
Nhận xét. Một cách khác để chứng minh bất đẳng thức \[\left ( \frac{a}{a+b} \right )^2+\left ( \frac{b}{b+c} \right )^2+\left ( \frac{c}{c+a} \right )^2 \ge \frac{3}{4}.\] Sử dụng bất đẳng thức Cauchy-Schwarz, ta có \[\left [ \sum \left ( \frac{a}{a+b} \right )^2 \right ]\left [ \sum (a+b)^2(a+c)^2 \right ]\ge \left [ \sum a(a+c) \right ]^2.\] Tuy nhiên \[4\left [ \sum a(a+c) \right ]^2 = \left [ \sum (a+b)^2 \right ]^2\] Vậy, ta chỉ cần chứng minh \[\left [ \sum (a+b)^2 \right ]^2 \ge 3 \sum (a+b)^2(a+c)^2,\] nhưng bất đẳng thức này đúng theo bất đẳng thức cơ bản \[(x+y+z)^2\ge 3(xy+yz+zx).\] Bất đẳng thức  được chứng minh.

 

Ngoài ra ta có thể chứng minh bài toán bằng cách sử dụng bất đẳng thức China TST 2004 của giáo sư Vasile Cirtoaje

\[\frac{a^2}{(a+b)^2}+\frac{b^2}{(b+c)^2}+\frac{c^2}{(c+d)^2}+\frac{d^2}{(d+a)^2} \geqslant 1.\]

 

 

Bài 115 (CĐTMO 2006) : Chứng minh rằng với mọi số thực $x,y,z$ thuộc đoạn $[1;2]$ ta luôn có bất đẳng thức :
                                          $(x+y+z)(\frac{1}{x}+\frac{1}{y}+\frac{1}{z})\geq 6(\frac{x}{y+z}+\frac{y}{x+z}+\frac{z}{x+y}).$

                                           Hỏi đẳng thức xảy ra khi và chỉ khi nào ?
 

 

Đặt \[f(x,\,y,\,z)=(x+y+z)\left( \frac{1}{x}+\frac{1}{y}+\frac{1}{z} \right)-6\left( \frac{x}{y+z}+\frac{y}{z+x}+\frac{z}{x+y} \right),\]

ta sẽ chứng minh $f(x,\,y,\,z)\ge 0$ bằng kỹ thuật dồn biến. Thật vậy, ta có $f(x,\,y,\,z)-f\left ( x,\,\frac{y+z}{2},\, \frac{y+z}{2}\right ),$ sẽ bằng \[\frac{(x+y+z)(y-z)^2}{yz(y+z)}-\frac{6(x+y+z)(y-z)^2}{(x+y)(x+z)(2x+y+z)},\] hay \[\frac{(x+y+z)(y-z)^2[(x+y)(z+x)(2x+y+z)-6yz(y+z)]}{(x+y)(y+z)(z+x)(2x+y+z)}.\] Giả sử $x$ là số lớn nhất trong ba số thì $(x+y)(z+x)(2x+y+z) > 6yz(y+z),$ nên $f(x,\,y,\,z)\ge f(x,\,t,\,t)$ với $t=\frac{y+z}{2} \ge 1.$

 

Tiếp đến, ta sẽ chứng minh $f(x,\,t,\,t)\ge 0.$ Bất đẳng thức tương đương với

$$(x+2t)\left( \frac{1}{x}+\frac{2}{t} \right)-6\left( \frac{x}{2t}+\frac{2t}{t+x} \right)\ge 0,$$

$$\frac{{{(t-x)}^{2}}(2t-x)}{tx(t+x)}\ge 0,$$

hiển nhiên đúng vì $2t\ge 2\ge x.$ Đẳng thức xảy ra khi và chỉ khi $x=y=z,$ hoặc $x=2$ và $y=z=1.$ Bài toán được chứng minh.

 

Nhận xét. Nếu $x,\,y,\,z$ là độ dài ba cạnh của tam giác thì bài toán vẫn đúng. Thật vậy, vì $x,\,y,\,z$ là độ dài ba cạnh của tam giác nên, theo phép thế Ravi, ta có thể viết bất đẳng thức lại như sau \[(x+y+z)\sum \frac{1}{x+y} \ge 3\sum \frac{y+z}{2x+y+z},\] tương đương với \[2\sum x^2(x^2-y^2)(x^2-z^2)+3\sum yz(y^2-z^2)^2 \ge 0,\] hiển nhiên đúng theo bất đẳng thức Shur.

 

Tác giả bài toán cũng là thầy Nam Dũng.


Bài viết đã được chỉnh sửa nội dung bởi Nguyenhuyen_AG: 19-06-2015 - 13:24

Nguyen Van Huyen
Ho Chi Minh City University Of Transport

#315
Hoang Tung 126

Hoang Tung 126

    Thiếu tá

  • Thành viên
  • 2061 Bài viết

Bài 115 (CĐTMO 2006) : Chứng minh rằng với mọi số thực $x,y,z$ thuộc đoạn $[1;2]$ ta luôn có bất đẳng thức :
                                          $(x+y+z)(\frac{1}{x}+\frac{1}{y}+\frac{1}{z})\geq 6(\frac{x}{y+z}+\frac{y}{x+z}+\frac{z}{x+y}).$

                                           Hỏi đẳng thức xảy ra khi và chỉ khi nào ? 

 

  Không mất tổng quát giả sử $x\geq y\geq z$ .Do $2\geq x,y,z\geq 1= > \left\{\begin{matrix} x+y\geq z & & \\ y+z\geq x & & \\ z+x\geq y & & \end{matrix}\right.$

 

BĐT $< = > (\sum x)(\sum \frac{1}{x})\geq 6(\sum \frac{x}{y+z})< = > (\sum x)(\sum \frac{1}{x})-9\geq 6(\sum \frac{x}{y+z}-\frac{3}{2})$

 

 Ta có :$(\sum x)(\sum \frac{1}{x})-9=\sum \frac{(y-z)^2}{yz}$

            $\sum \frac{x}{y+z}-\frac{3}{2}=\sum \frac{(y-z)^2}{2(y+x)(z+x)}$

 

 Do đó BĐT $< = > \sum \frac{(y-z)^2}{yz}\geq 6(\sum \frac{(y-z)^2}{2(y+x)(z+x)})$

$< = > \sum \frac{(y-z)^2}{yz}\geq 3\sum \frac{(y-z)^2}{(y+x)(z+x)}< = > \sum (y-z)^2(\frac{1}{yz}-\frac{3}{(y+x)(z+x)})\geq 0$

$< = > (x-y)^2(\frac{1}{xy}-\frac{3}{(x+z)(y+z)})+(y-z)^2(\frac{1}{yz}-\frac{3}{(y+x)(z+x)})+(x-z)^2(\frac{1}{xz}-\frac{3}{(x+y)(z+y)})\geq 0$

 

  Đặt $\left\{\begin{matrix} S_{a}=\frac{1}{yz}-\frac{3}{(y+x)(z+x)} & & \\ S_{b}=\frac{1}{xz}-\frac{3}{(x+y)(z+y)} & & \\ S_{c}=\frac{1}{xy}-\frac{3}{(x+z)(y+z)} & & \end{matrix}\right.$

 

Ta có :$S_{b}=\frac{1}{xz}-\frac{3}{(x+y)(z+y)}=\frac{xy+yz+y^2-2xz}{xz(x+y)(z+y)}=\frac{x(y-z)+y(y+z)-xz}{xz(x+y)(z+y)}\geq \frac{x(y-z)+xy-xz}{xz(x+y)(z+y)}=\frac{2x(y-z)}{xz(x+y)(z+y)}=\frac{2(y-z)}{z(x+y)(z+y)}\geq 0= > S_{b}\geq 0$

  (Do $y+z\geq x= > y(y+z)\geq xy,y\geq z= > y-z\geq 0$)

          $S_{a}=\frac{1}{yz}-\frac{3}{(x+y)(x+z)}=\frac{x^2+xy+xz-2yz}{yz(x+y)(x+z)}=\frac{x^2+y(x-z)+z(x-y)}{yz(x+y)(x+z)}> 0= > S_{a}> 0$ (Do $x\geq z= > y(x-z)\geq 0,x\geq y= > z(x-y)\geq 0,x^2> 0= > x^2+y(x-z)+z(x-y)> 0$)

  Do đó $S_{a}+S_{b}> 0$

 

Ta có :$S_{b}+S_{c}=\frac{1}{xz}+\frac{1}{xy}-\frac{3}{(x+y)(z+y)}-\frac{3}{(x+z)(y+z)}$

$=\frac{y+z}{xyz}-\frac{3(x+z)+3(x+y)}{(x+y)(y+z)(x+z)}=\frac{y+z}{xyz}-\frac{6x+3y+3z}{(x+y)(y+z)(x+z)}$

  Ta chứng minh $S_{b}+S_{c}\geq 0< = > \frac{y+z}{xyz}\geq \frac{6x+3y+3z}{(x+y)(y+z)(x+z)}< = > (x+y)(x+z)(y+z)^2\geq xyz(6x+3y+3z)< = > (x^2+xz+xy+yz)(y^2+2yz+z^2)\geq 6x^2yz+3xy^2z+3xyz^2< = > x^2y^2+2x^2yz+x^2z^2+xy^2z+2xyz^2+xz^3+xy^3+2xy^2z+xyz^2+y^3z+2y^2z^2+yz^3\geq 6x^2yz+3xy^2z+3xyz^2< = > x^2y^2+x^2z^2+y^3z+yz^3+2y^2z^2+xz^3+xy^3\geq 4x^2yz$

               $< = > yz(y+z)^2+x^2(y^2+z^2)+x(y^3+z^3)\geq 4x^2yz$

  BĐT này đúng do theo Cosi ta có :

   $x^2(y^2+z^2)\geq x^2.2yz=2x^2yz$

   $x(y^3+z^3)\geq x.yz(y+z)\geq x^2yz$ ( Do $y+z\geq x$)

   $yz(y+z)^2\geq x^2yz$ (Do $y+z\geq x$

 

Cộng theo vế $= > yz(y+z)^2+x^2(y^2+z^2)+x(y^3+z^3)\geq 4x^2yz$ (Luôn đúng)

 

   Do đó $S_{b}+S_{c}\geq 0$

 

Từ đó $S_{b}\geq 0,S_{b}+S_{a}> 0,S_{b}+S_{c}\geq 0$ nên theo nguyên lý SOS thì BĐT đề bài được chứng minh. 

 

    Dấu = xảy ra khi $x=y=z$ hoặc $x=2y=2z$ và hoán vị của chúng


Bài viết đã được chỉnh sửa nội dung bởi Hoang Tung 126: 19-06-2015 - 13:21


#316
dangkhuong

dangkhuong

    Sĩ quan

  • Thành viên
  • 312 Bài viết

Bài 113:(Palestine MO): Cho a,b,c>0. Cmr:

 

$\frac{(a^3+2abc)^3}{(2a^3+c^3)^2}+\frac{(b^3+2abc)^3}{(2b^3+c^3)^2}+\frac{(c^3+2abc)^3}{(2c^3+a^3)^2}\leq 3(a^3+b^3+c^3)$

sau đây là lời giải cho bài 113:(vắn tắt như sau)

 

1) theo bất đẳng thức Holder thì $(a^3+b^3+b^3)(a^3+a^3+c^3)(a^3+a^3+c^3)\geq (a^3+bc^2+ba^2)^3=(a^3+b(a^2+c^2))^3\geq (a^3+2abc)^3$

 Do đó ta có $\frac{(a^3+2abc)^3}{(2a^3+c^3)^2}\leq a^3+2b^3$

2)Tương tự ta có các bất đẳng thức sau:

 $\frac{(b^3+2abc)^3}{(2b^3+c^3)^2}\leq b^3+2c^3$

$\frac{(c^3+2abc)^3}{(2c^3+a^3)^2}\leq c^3+2a^3$

 Cộng ba vế bất đẳng thức trên ta có đpcm.(lời giải của tác giả thì mình ko bit đây là lời giải của mình)


Bài viết đã được chỉnh sửa nội dung bởi dangkhuong: 19-06-2015 - 15:51

:ukliam2:  :ukliam2:  :ukliam2:


#317
dangkhuong

dangkhuong

    Sĩ quan

  • Thành viên
  • 312 Bài viết

Bài 118(Sweeden MO): Cho a,b,c>0. Cmr nếu a+b+c=3 thì 

      $\sqrt[5]{\frac{a^3+b^3+1}{b^2+2c^2}}+\sqrt[5]{\frac{b^3+c^3+1}{c^2+2a^2}}+\sqrt[5]{\frac{c^3+a^3+1}{a^2+2b^2}}\geq \frac{9abc}{ab+bc+ca}$


Bài viết đã được chỉnh sửa nội dung bởi dangkhuong: 19-06-2015 - 16:06

:ukliam2:  :ukliam2:  :ukliam2:


#318
dangkhuong

dangkhuong

    Sĩ quan

  • Thành viên
  • 312 Bài viết

Bài 119(Thailand MO): Cho a,b,c>0. CMR: 

$\frac{a^2}{b^2+bc+c^2}+\frac{b^2}{c^2+ca+a^2}+\frac{c^2}{a^2+ab+b^2}\geq 1$

 


Bài viết đã được chỉnh sửa nội dung bởi dangkhuong: 19-06-2015 - 16:09

:ukliam2:  :ukliam2:  :ukliam2:


#319
Belphegor Varia

Belphegor Varia

    Thượng sĩ

  • Thành viên
  • 227 Bài viết

Bài 119(Thailand MO): Cho a,b,c>0. CMR: :ukliam2:  :ukliam2:  :ukliam2: 

$\frac{a^2}{b^2+bc+c^2}+\frac{b^2}{c^2+ca+a^2}+\frac{c^2}{a^2+ab+b^2}\geq 1$

 

(bài này cực hay....)

Spam : Mình xin góp ý là bạn chỉ nên post đề bài để mọi người cùng thảo luận là đủ  ^_^ , không nên để thêm mấy icon hoặc lời nhắn không cần thiết cạnh đề bài


$ \textbf{NMQ}$

Wait a minute, You have enough time. Also tomorrow will come 

Just take off her or give me a ride 

Give me one day or one hour or just one minute for a short word 

 


#320
dangkhuong

dangkhuong

    Sĩ quan

  • Thành viên
  • 312 Bài viết

Xin trình bày lời giải bài số 119 như sau:

 

Không mất tính tổng quát ta giả sử rằng $a\geq b\geq c$

 

khi đó ta dễ thấy rằng: $a^2\geq b^2\geq c^2$ ; $\frac{1}{b^2+c^2+bc}\geq \frac{1}{c^2+ca+a^2}\geq \frac{1}{a^2+ab+b^2}$

 

Áp dụng bất đẳng thức Chebysev,và bất đẳng thức AM-GM ta có:

$\frac{a^2}{b^2+bc+c^2}+\frac{b^2}{c^2+ca+a^2}+\frac{c^2}{a^2+ab+b^2}\geq \frac{1}{3}(a^2+b^2+c^2)(\sum \frac{1}{b^2+bc+c^2})\geq 3(a^2+b^2+c^2)(\frac{1}{2(a^2+b^2+c^2)+ab+bc+ca})\geq 3(a^2+b^2+c^2)(\frac{1}{3(a^2+b^2+c^2)})=1$

 

Vậy là ta có đpcm. :ukliam2:  :ukliam2:  :ukliam2: 


:ukliam2:  :ukliam2:  :ukliam2:





0 người đang xem chủ đề

0 thành viên, 0 khách, 0 thành viên ẩn danh